For a set of vectors to span R4 question

  • Thread starter Thread starter Samuelb88
  • Start date Start date
  • Tags Tags
    Set Span Vectors
Click For Summary
The set of vectors u, v, and w is linearly dependent and cannot span R4 on its own. To create a spanning set for R4, at least two additional linearly independent vectors must be added. The Gram-Schmidt process can be utilized to generate a new set of orthogonal vectors from u, v, and w, allowing for the selection of a fourth vector. It is essential to ensure that the final set contains four linearly independent vectors to span R4. Therefore, simply adding one more vector to the existing set is insufficient; two or more additional vectors are necessary.
Samuelb88
Messages
160
Reaction score
0

Homework Statement


The set of vectors u = {1,-2,2,1}, v = {1,3,1,1}, w = {3,4,4,3} cannot span R4. Complete this set to create a set of vectors that will span R4. Show that your set of vectors spans R4.

The Attempt at a Solution


Let y = {y_1,y_2,y_3,y_4}. I write span{u,v,w,y} as the coefficient matrix:

[1,1,3,y_1
-2,3,4,y_2
2,1,4,y_3
1,1,3,y_4]

Using the first row to produce zeros in each row below yields:

[1,1,3,y_1
0,5,10,y_2+2y_1
0,-1,-2,y_3-2y_1
0,0,0,y_4-y_1]

Using the second row to produce zeros in the row below yields:

[1,1,3,y_1
0,5,10,y_2+2y_1
0,0,0,5y_3-9y_2+2y_1
0,0,0,y_4-y_1]

So to my understanding, it would seem given the set {u,v,w}, a fourth vector y cannot be chosen so that the set {u,v,w,y} spans R4 since not every row can contain a pivot position in this case. Please correct me if I am wrong.

- Sam

PS sorry for the messy work - I don't know how to write matrices in latex.
 
Physics news on Phys.org
The problem is that u, v, and w are linearly dependent. Have you learned about the Gram-Schmidt process for constructing a basis yet?
 
Mark44 said:
The problem is that u, v, and w are linearly dependent. Have you learned about the Gram-Schmidt process for constructing a basis yet?

No I haven't. To my understanding though, the Gram-Schmidt process creates a set (for this case) of vectors {u',v',w'} so that v' is orthogonal to u', w' is orthogonal to v' and I suppose u', v', and w' are linearly independent so I may choose a fourth vector y' to "complete" this set so that this completed set spans R4?
 
You can use G-S to construct all four vectors of a basis for R4.
 
Samuelb88 said:
So to my understanding, it would seem given the set {u,v,w}, a fourth vector y cannot be chosen so that the set {u,v,w,y} spans R4 since not every row can contain a pivot position in this case. Please correct me if I am wrong.
You're right. You can't add just one more vector and span R4. You need to add two or more because, as Mark noted, the three you have are linearly dependent.
PS sorry for the messy work - I don't know how to write matrices in latex.
You can use the bmatrix environment to produce matrices in LaTeX. Click on the one below to see what the code is.

\begin{bmatrix}<br /> 1 &amp; 1 &amp; 3 &amp; y_1 \\<br /> -2 &amp; 3 &amp; 4 &amp; y_2 \\<br /> 2 &amp; 1 &amp; 4 &amp; y_3 \\<br /> 1 &amp; 1 &amp; 3 &amp; y_4<br /> \end{bmatrix}
 
Don't include 0. Your u, v, and w are linearly dependent, which can be seen from the first 3 columns of your last matrix (but taken one step further).

\begin{bmatrix}1 &amp; 0 &amp; 1 \\0 &amp; 1 &amp; 2 \\0 &amp; 0 &amp; 0 \\0 &amp; 0 &amp; 0\end{bmatrix}

This says c1 + c3 = 0, and that c2 + 2c3= 0, where c1, cw, and c3 are the coefficients of u, v, and w. If you let c3 = 1, then c1 = -1, c2 = -2, and c3 = 1.

In other words -u -2v + w = 0, so it can be seen that anyone of these vectors can be solved for in terms of the other two.

Without knowing G-S, I suppose the best way is to pick two of the three vectors, say u and v, and then try to find two more vectors out of the set of standard basis vectors for R4 - {<1, 0, 0, 0>, <0, 1, 0, 0>, <0, 0, 1, 0>, <0, 0, 0, 1>} - so that you have four linearly independent vectors.

For your spanning set you can throw in w.
 
Question: A clock's minute hand has length 4 and its hour hand has length 3. What is the distance between the tips at the moment when it is increasing most rapidly?(Putnam Exam Question) Answer: Making assumption that both the hands moves at constant angular velocities, the answer is ## \sqrt{7} .## But don't you think this assumption is somewhat doubtful and wrong?

Similar threads

  • · Replies 8 ·
Replies
8
Views
2K
  • · Replies 2 ·
Replies
2
Views
2K
  • · Replies 6 ·
Replies
6
Views
3K
  • · Replies 8 ·
Replies
8
Views
2K
  • · Replies 3 ·
Replies
3
Views
2K
  • · Replies 8 ·
Replies
8
Views
2K
  • · Replies 5 ·
Replies
5
Views
2K
  • · Replies 5 ·
Replies
5
Views
3K
  • · Replies 3 ·
Replies
3
Views
2K
  • · Replies 9 ·
Replies
9
Views
5K